Vous êtes sur la page 1sur 3

1) A rectangular coil of wire, of width 22.0cm and height 35.0cm and carrying a current of 1.

40A, is
oriented with the plane of its loop parallel to a uniform magnetic field of 1.50T, as shown in the
figure (Note: The current is going anticlockwise around the wire and the magnetic field is going
to the left of the page)

a. What is the force exerted on each vertical section of wire for this situation - (the left
vertical part and the the right vertical part)? (2 marks)

𝐹 = 𝐵𝐼𝐿 = 1.5 × 1.40 × 35 × 10 = 𝟎. 𝟕𝟑𝟓𝑵 (𝑺𝒂𝒎𝒆 𝒇𝒐𝒓 𝒃𝒐𝒕𝒉 𝒔𝒊𝒅𝒆𝒔)

Correct use of formula (1), Correct Answer (1) – ANSWERS MUST NOT CONTRADICT

b. What direction do the forces point on each vertical part of the wire? (you can describe
them or label them on the diagram above) (2 marks)

Left part- Into the page


Right part- out of the page

The answer must be very clear. E.g, I will not accept ‘out’, but will accept ‘coming out and hitting you in
the face if you look at it face on’
c. What is the force exerted on each horizontal section of wire for this situation? (1 mark)

Zero

d. Why? (1 mark)

The current (or horizontal section of the wire) runs parallel to the magnetic field here/’it’ isn’t
acceptable

e. What is the resultant translational force on the whole coil for this situation? (1 mark)
Zero
f. What is the torque on the coil for this situation? (2 marks)

𝜏 = 𝑁. 𝐵𝐼𝐴 = 1.5 × 1.4 ×× (35 × 10 ) × (22 × 10 ) = 𝟎. 𝟏𝟔𝟐𝑵𝒎


Correct use of formula (1), Correct Answer (1)

g. What position does the coil need to be in to give a torque of zero on the coil? (Be VERY
clear!!!) (1 mark)

The plane of the coil must be perpendicular to (the plane) of the magnetic field OR
equivalent

Only a very clear answer or diagram gets the mark

h. Why is the torque zero for the position you described in your answer to part ‘g’? (Be
VERY clear!!!) (1 mark)

The perpendicular distance between the force and pivot is zero/There is zero distance
contributing to the moment OR equivalent - NEED to reference that it’s the distance
from the force

Only a very clear answer gets the mark

i. The coil is rotated through a 45.0o angle about the fixed axis shown in the diagram
above by the vertical line, so that the left vertical part is going into the plane of the
paper and the right vertical part is coming out of the plane of the paper. What is the
force on each vertical part of the wire for this situation? (1 mark)
0.735N (1 mark) – force is constant!!!

j. What is the torque on the coil for the situation described above in part ‘i’? (2 marks)

𝜏 = 𝐵𝐼𝐴𝑆𝑖𝑛𝜃 = 1.5 × 1.4 ×× (35 × 10 ) × (22 × 10 ) × 𝑆𝑖𝑛45 = 𝟎. 𝟏𝟏𝟒𝑵𝒎

Correct use of formula (1), Correct Answer (1)

k. How can we make the torque on a coil in a magnetic field the highest value possible?
(Answer clearly: Refer to equations, quantities, vectors and the particle’s motion) (6
marks)
 Clarity of explanation (1)
 Clear mention and reference to 𝜏 = (𝑁)𝐵𝐼𝐴𝑆𝑖𝑛𝜃 (1)
 The plane of the magnetic field must match the plane of the loop/The loop must
be in a position that the angle gives a maximum perpendicular distance for the
moment on each arm/angle must be 90 degrees (with a clear reference to the
coil’s position) – THIS MUST BE VERY CLEAR (1)
 Increase the area of the loop (Increase height or width) (1)
 Increase the current ‘I’ through the loop (1)
 Increase the magnetic field strength (1)
 Change the shape of wire to a circle (1)
 Increase the number of turns in the coil ‘N’ (1)
 Note: if you just listed it with no sound attempt at explanation, e.g “Increase,
N, I, B, A” – I deducted marks because that’s not explaining and it doesn’t
demonstrate your knowledge

Vous aimerez peut-être aussi